3
$\begingroup$

Prove that there is no positive integer $C$ and the sequence of positive integers$ (a_n) $ satisfies : ${a_{k+1}}^k|C^k(a_1.a_2...a_k)$ for all $k\in Z+$ ($(a_n)$ is the sequence of numbers such that all numbers are DIFFERENT.)

Suppose there exists a positive integer $C$ and such a sequence $(a_n)$ .

Consider any prime $p$. Set $vp(C) = C' , vp(a_i)=b_i$

From the assumption $\Rightarrow kb_{k+1} \le kC'+b_1+b_2+...+b_k$

Consider the sequence $(v_n)$ as follows: $v_1=b_1 ; kv_{k+1} = kC'+v_1+v_2+...+v_k$ Thus, $b_k \le v_k$

$kv_{k+1}=kC'+v_1+v_2+...+v_k=C'+(k-1)C'+v_1+v_2+...+v_{k-1}+v_k = C'+ (k-1)v_k+v_k=C'+kv_k$ $\Rightarrow v_{k+1} = C'/k + v_k$

So if $k→+°$ , then because $b_k \in Z+$ $(b_n)$ is bounded on .

So $(b_n)$ is bounded above and below .

There should be an infinite $i$ such that there exists an infinite $j$ such that $b_i = b_j$

This is all I can do , this is a very difficult problem for me . Hope to get help from everyone. Thanks very much !

asked Feb 19, 2022 at 6:37
$\endgroup$
2
  • $\begingroup$ What if $C = 1$ and $a_k = 1$ for all positive integers $k$? $\endgroup$ Commented Feb 19, 2022 at 6:52
  • $\begingroup$ @VTand sorry so much! (a_n) is the sequence of numbers such that all numbers are DIFFERENT. Thank you ! $\endgroup$ Commented Feb 19, 2022 at 6:56

1 Answer 1

0
$\begingroup$

You're asking about proving there's no positive integer $C$ and an infinite set of distinct positive integers $a_i$ such that

$$(a_{k+1})^k \mid C^k\left(\prod_{i=1}^{k}a_i\right) \tag{1}\label{eq1A}$$

is true for all positive integers $k$. Assume there is some such a positive integer $C$ and set of $a_i$. For any prime $p_j$ which divides $C$ or one of the $a_i$, using the $p$-adic order function, let

$$b_{j,i} = \nu_{p_{j}}(a_i), \; \; c_j = \nu_{p_{j}}(C) \tag{2}\label{eq2A}$$

We'll prove using induction, with the harmonic numbers (i.e., $H_1 = 1$ and $H_{n+1} = H_n + \frac{1}{n+1} \; \forall \; n \ge 1$), for all $k \ge 1$ that

$$kb_{j,k+1} \le kc_j + \sum_{i=1}^{k}b_{j,i} \le kH_{k}c_j + kb_{j,1} \; \; \to \; \; b_{j,k+1} \le H_{k}c_j + b_{j,1} \tag{3}\label{eq3A}$$

For $k = 1$, we get from \eqref{eq1A} and \eqref{eq2A} that

$$b_{j,2} \le c_{j} + b_{j,1} = H_{1}c_{j} + b_{j,1} \tag{4}\label{eq4A}$$

so \eqref{eq3A} works in the base case since $H_1 = 1$. Assume \eqref{eq3A} is true for $k = n$ for some $n \ge 1$. Then, for $k = n + 1$, \eqref{eq1A}, \eqref{eq2A} and \eqref{eq3A} give that

$$\begin{equation}\begin{aligned} (n+1)b_{j,n+2} & \le (n + 1)c_j + \sum_{i=1}^{n+1}b_{j,i} \\ & = \left({\color{blue}{nc_j + \sum_{i=1}^{n}b_{j,i}}}\right) + c_j + {\color{green}{b_{j,n+1}}} \\ & \le ({\color{blue}{nH_{n}c_j + nb_{j,1}}}) + c_j + ({\color{green}{H_{n}c_j + b_{j,1}}}) \\ & = ((n+1)H_{n} + 1)c_j + (n + 1)b_{j,1} \\ & = (n + 1)\left(H_{n} + \frac{1}{n+1}\right)c_j + (n + 1)b_{j,1} \end{aligned}\end{equation}\tag{5}\label{eq5A}$$

Dividing both sides of \eqref{eq5A} by $n + 1$ and using $H_{n+1} = H_{n} + \frac{1}{n+1}$ gives

$$b_{j,n+2} \le \left(H_{n} + \frac{1}{n+1}\right)c_j + b_{j,1} = H_{n+1}c_j + b_{j,1} \tag{6}\label{eq6A}$$

so \eqref{eq3A} holds for $k = n + 1$ as well. Thus, by induction, \eqref{eq3A} holds for all positive integers $k$.

Note \eqref{eq1A} shows each prime factor $p_j$ of any $a_{k+1}$ must have $p_j \mid C$ and/or $p_j \mid a_i$ for some $i \le k$. If $p_j \not\mid C$, so $p_j \mid a_i$ $i \gt 1$, then using \eqref{eq1A} again shows that $p_j \mid a_q$ for some $q \lt i$. This can continue until we get $p_j \mid a_1$. Thus, we have that all prime factors of any $a_{k+1}$ divides $Ca_1$, so there are only a finite number of them, say some $r \ge 1$. Also, the harmonic numbers being a strictly increasing sequence means, from \eqref{eq3A}, that $b_{j,i} \le H_{k}c_j + b_{j,1}$ for all 1ドル \le i \le k + 1$. Thus, considering all of the $r$ prime factors being used, we have that all $a_i$ for 1ドル \le i \le k + 1$ must be factors of

$$M_k = \prod_{j=1}^{r}p_j^{e_j}, \; e_j = \lfloor H_{k}c_{j} \rfloor + b_{j,1} \tag{7}\label{eq7A}$$

Thus, using that the $a_i$ for 1ドル \le i \le k + 1$ comprise $k + 1$ distinct positive factors of $M_k$, the number-of-divisors function $\sigma_0()$, an upper bound for the harmonic numbers growth rate of $H_k \lt \ln k + 1$ (for $k \ge 2$, from $H_k = \ln k + \gamma + \frac{1}{2k} - \varepsilon_k$ with $\gamma \approx 0.5772$ and $\varepsilon_k \ge 0$) and $c_{m} = \max(c_{1} + 1, \ldots, c_{r} + 1)$, we get for large enough $k$ (i.e., where $\ln k \gt \max(c_{1} + b_{1,1} + 1, \ldots, c_{r} + b_{r,1} + 1)$) that

$$\begin{equation}\begin{aligned} k + 1 & \le \sigma_0(M_k) \\ & = \prod_{j=1}^{r}(\lfloor H_{k}c_{j} \rfloor + b_{j,1} + 1) \\ & \lt \prod_{j=1}^{r}(c_{j}\ln{k} + c_{j} + b_{j,1} + 1) \\ & \lt \prod_{j=1}^{r}((c_{j} + 1)\ln{k}) \\ & \le \prod_{j=1}^{r}(c_{m}\ln{k}) \\ & = c_{m}^{r}(\ln{k})^{r} \end{aligned}\end{equation}\tag{8}\label{eq8A}$$

For some real $x$ with $k = e^x$, we have that $k \to \infty$ means $x \to \infty$. Also, \eqref{eq8A} becomes

$$e^x + 1 \lt c_{m}^{r}x^{r} \tag{9}\label{eq9A}$$

However, exponential functions grow faster than any polynomial (e.g., as can been seen for our case by using L'Hôpital's rule $r$ times to get $\lim_{x \to \infty}\frac{e^x + 1}{c_{m}^{r}x^{r}} = \lim_{x \to \infty}\frac{e^x}{c_{m}^{r}(r!)} = \infty$, or by using a post like Proving exponential is growing faster than polynomial), so \eqref{eq9A} can't be true for large enough $x$ and, thus, large enough $k$.

This contradicts the assumption that such a $C$ and set of $a_i$ exists, proving that \eqref{eq1A} can't always be true.

answered Feb 19, 2022 at 23:08
$\endgroup$

You must log in to answer this question.

Start asking to get answers

Find the answer to your question by asking.

Ask question

Explore related questions

See similar questions with these tags.